Piece of cake

Algebra Level 5

10 ( a 3 + b 2 + c ) \large 10(a^3+b^2+c) If a , b a,b and c c are positive reals satisfying a b c = 1 abc=1 , find the minimum value of the expression above. Round your answer to the nearest integer.

Part of the set


The answer is 27.

This section requires Javascript.
You are seeing this because something didn't load right. We suggest you, (a) try refreshing the page, (b) enabling javascript if it is disabled on your browser and, finally, (c) loading the non-javascript version of this page . We're sorry about the hassle.

2 solutions

Department 8
Feb 5, 2016

@Gurīdo Cuong , please tell me whether I did same as yours?

By AM-GM, we have

a 3 2 + a 3 2 + b 2 3 + b 2 3 + b 2 3 + c 6 + c 6 + c 6 + c 6 + c 6 + c 6 11 ( a b c ) 6 2 2 × 3 3 × 6 6 11 10 ( a 3 + b 2 + c ) 110 1 2 8 × 3 9 11 27.045 \large{\frac { { a }^{ 3 } }{ 2 } +\frac { { a }^{ 3 } }{ 2 } +\frac { { b }^{ 2 } }{ 3 } +\frac { { b }^{ 2 } }{ 3 } +\frac { { b }^{ 2 } }{ 3 } +\frac { c }{ 6 } +\frac { c }{ 6 } +\frac { c }{ 6 } +\frac { c }{ 6 } +\frac { c }{ 6 } +\frac { c }{ 6 } \ge 11\sqrt [ 11 ]{ \frac { { \left( abc \right) }^{ 6 } }{ { 2 }^{ 2 }\times { 3 }^{ 3 }\times { 6 }^{ 6 } } } \\ 10\left( { a }^{ 3 }+{ b }^{ 2 }+c \right) \ge 110\sqrt [ 11 ]{ \frac { 1 }{ { 2 }^{ 8 }\times { 3 }^{ 9 } } } \approx 27.045}

This means the minimum value of 10 ( a 3 + b 2 + c ) 10\left( { a }^{ 3 }+{ b }^{ 2 }+c \right) is approximately 27. Equality holds when a 3 2 = b 2 3 = c 6 \frac { { a }^{ 3 } }{ 2 } =\frac { { b }^{ 2 } }{ 3 } =\frac { c }{ 6 }

Nice solution(+1).. But aren't a,b,c fixed in case of equality i.e equality occurs when: a = 2 3 ( 1 6 3 ( 2 ) 1 3 ) 2 11 b = 3 ( 1 6 3 ( 2 ) 1 3 ) 3 11 a n d c = 6 ( 1 6 3 ( 2 ) 1 3 ) 6 11 a=\sqrt[3]{2}(\dfrac{1}{6\sqrt3(2)^{\small{\frac{1}{3}}}})^\frac{2}{11}\\ b=\sqrt{3}(\dfrac{1}{6\sqrt3(2)^{\small{\frac{1}{3}}}})^\frac{3}{11}\\ and~ c=6(\dfrac{1}{6\sqrt3(2)^{\small{\frac{1}{3}}}})^\frac{6}{11}

Rishabh Jain - 5 years, 4 months ago

Log in to reply

If I remember correctly equality holds in AM-GM when all terms are equal to each other here a 3 2 = b 2 3 = c 6 \frac{ a ^{ 3 } }{ 2 } =\frac { { b }^{ 2 } }{ 3 } =\frac { c }{ 6 } and if you look carefully you will get the same result and you have just given me precise value of them.

Department 8 - 5 years, 4 months ago

Log in to reply

You have found the condition for equality as a 3 2 = b 2 3 = c 6 \frac{ a ^{ 3 } }{ 2 } =\frac { { b }^{ 2 } }{ 3 } =\frac { c }{ 6 } and given condition is a b c = 1 abc =1 . Combining both, a,b,c are fixed.

Rishabh Jain - 5 years, 4 months ago

Log in to reply

@Rishabh Jain Duh, bro it's a b c = 1 abc=1 look the question carefully

Department 8 - 5 years, 4 months ago

Log in to reply

@Department 8 Just a typo.. : { Fixed...Now what say??

Rishabh Jain - 5 years, 4 months ago

Log in to reply

@Rishabh Jain Multiply them and you will see it's equal to 1

Department 8 - 5 years, 4 months ago

same way buddy.I guess both ironman and batman think the same way most of the time

Kaustubh Miglani - 5 years, 4 months ago

Log in to reply

Working against wrong guys with technologically advanced things require the same skills.

Department 8 - 5 years, 4 months ago

Exactly Same Way

Kushagra Sahni - 5 years, 4 months ago

Same way. Why are you not coming to class

Shreyash Rai - 5 years, 4 months ago

Log in to reply

Test series was going in school.

Department 8 - 5 years, 4 months ago

i did same

Dev Sharma - 5 years, 4 months ago
P C
Feb 5, 2016

We assume that a = l a=l and b = k b=k when the equality holds

Using AM-GM we have a 3 + 2 l 3 3 a l 2 a^3+2l^3\geq 3al^2 and b 2 + k 2 2 b k b^2+k^2\geq 2bk

So
a 3 + b 2 + c 3 a l 2 + 2 b k + c 2 l 3 k 2 3 6 l 2 k 3 2 l 3 k 2 a^3+b^2+c\geq 3al^2+2bk+c-2l^3-k^2\geq 3\sqrt[3]{6l^2k}-2l^3-k^2 Now k and l must satisfy { a = l b = k 3 a l 2 = 2 b k = c l k c = 1 \begin{cases} a=l\\b=k\\3al^2=2bk=c\\lkc=1\end{cases} Solving and we get { l = 2 27 11 k = 3 2 ( 2 27 ) 3 11 c = 3 ( 2 27 ) 3 11 \begin{cases} l=\sqrt[11]{\frac{2}{27}}\\k=\sqrt{\frac{3}{2}\sqrt[11]{(\frac{2}{27})^3}}\\c=3\sqrt[11]{(\frac{2}{27})^3}\end{cases} So the minimum of the expression is 27.045... \approx 27.045... which is 27 27 when rounded to the nearest integer

0 pending reports

×

Problem Loading...

Note Loading...

Set Loading...